Función inversa f−1:f(R)→Rf−1:f(R)→Rf^{-1}:f(\mathbb{R})\to\mathbb{R} de una función estrictamente creciente f:R →Rf:R→Rf:\mathbb{R}\to\mathbb{R} es continua

He probado la siguiente afirmación y me gustaría saber si mi prueba es correcta y/o/si/cómo se puede mejorar.

"Suponer F : R R es una función estrictamente creciente.

Demostrar que la función inversa F 1 : F ( R ) R es una función continua".

mi prueba:

Dejar F : R R sea ​​una función estrictamente creciente: entonces es inyectiva y como función F : R F ( R ) debe ser sobreyectiva por lo que tiene una inversa F 1 : F ( R ) R que debe ser estrictamente creciente también ( 1 ) .

Supongamos ahora que F 1 eran discontinuos en un punto y d F ( R ) : entonces, siendo una función creciente, y d debe ser una discontinuidad de salto por lo que el intervalo I y d := ( límite y y d ,   y F ( R ) F 1 ( y ) , límite y y d + ,   y F ( R ) F 1 ( y ) ) = ( sorber y < y d ,   y F ( R ) F 1 ( y ) , inf y > y d ,   y F ( R ) F 1 ( y ) ) debe ser no vacío y podemos escoger un elemento X ¯ X d = F 1 ( y d ) en eso asi F ( X ¯ ) = y d , pero siendo F estrictamente creciente por hipótesis también es F ( X ¯ ) > y d o F ( X ¯ ) < y d , una contradicción.

Entonces, F 1 ( R ) R no puede ser discontinuo en ningún punto, es decir, debe ser continuo en F ( R ) .


( 1 ) dejar y 1 , y 2 F ( R ) y supongamos wlog y 1 < y 2 : entonces F 1 ( y 1 ) = F 1 ( F ( X 1 ) ) = X 1 y F 1 ( y 2 ) = F 1 ( F ( X 2 ) ) = X 2 y si X 1 X 2 entonces F ( X 1 ) = y 1 y 2 = F ( X 2 ) contradicción, por lo que debe ser X 1 = F 1 ( y 1 ) < X 2 = F 1 ( y 2 )

¿La función f es continua?
@EduardoMaza no, no es continuo
@MartinR gracias por su interés en mi pregunta y por estos enlaces, he aprendido una nueva prueba de esta afirmación al leer la segunda. Sin embargo, me parece que mi prueba es un poco diferente de estas dos: ¿te importaría revisarla y decirme si crees que es correcta?
Tenga en cuenta que el límite del lado izquierdo y/o del lado derecho puede no existir, por ejemplo, si y d es un punto aislado de F ( R ) . Del mismo modo, los conjuntos { y < y d y F ( R ) } y/o { y > y d y F ( R ) } puede estar vacío, de modo que su supremum resp. el mínimo no está definido.
@Martin R gracias por tu comentario; con respecto a su primer comentario, siendo F 1 aumentando, ¿no debería existir siempre el límite de los lados izquierdo y derecho ( math.stackexchange.com/q/4171854 ) (incluso si y d es un punto aislado, consulte math.stackexchange.com/questions/27429/… )?

Respuestas (3)

Considere la función F 1 : R R dada por

F 1 ( y ) := { y 1 y < 0 y y 0.
Aquí, uso la etiqueta " F 1 "solo formalmente, no como una indicación de que es el inverso de alguna función F (aunque, por supuesto, es la inversa de una función).

Claramente, F 1 es estrictamente creciente, y es una biyección del rango de su inversa a R . Alquiler y d = 0 , vemos eso

( límite y y d ,   y dom F 1 F 1 ( y ) , límite y y d + ,   y dom F 1 F 1 ( y ) ) = ( 1 , 0 )
ciertamente no está vacío, pero no contiene elementos del rango de F 1 --es decir, ningún elemento del dominio del inverso de F 1 .

Ese es el defecto de tu argumento. El hecho de que un intervalo no esté vacío no significa que contenga un elemento en el dominio de una función arbitraria estrictamente creciente. En otras palabras, en realidad no ha justificado la siguiente afirmación.

podemos elegir un elemento X ¯ X d = F 1 ( y d ) en [el intervalo no vacío] entonces F ( X ¯ ) = y d

Dado que todo lo que has concluido sobre F 1 es que es estrictamente creciente y es una biyección del rango de su inversa a R , entonces es muy posible que F 1 = gramo , en cuyo caso su argumento se cae.

Agregado : un mejor enfoque sería proceder directamente. Tome un arbitrario y 0 F [ R ] , y deja X 0 := F 1 ( y 0 ) . Desde F es estrictamente creciente, entonces para X < X 0 (resp., por X > X 0 ) tenemos F ( X ) < y 0 (resp. F ( X ) > y 0 ).

Tome un arbitrario ε > 0 , dejar y metro := F ( X 0 ε ) , y deja y METRO := F ( X 0 + ε ) , de modo que y metro , y METRO F [ R ] y y metro < y 0 < y METRO .

Alquiler d = min { y 0 y metro , y METRO y 0 } , tenemos d > 0 , y para todos y R , si | y y 0 | < d , entonces y metro < y < y METRO .

En particular, tome cualquier y F [ R ] tal que | y y 0 | < d , y deja X = F 1 ( y ) . Desde F es estrictamente creciente y F ( X 0 ε ) = y metro < y = F ( X ) , entonces X 0 ε < X . Similarmente, X < X 0 + ε , y entonces | X X 0 | < ε , o equivalente,

| F 1 ( y ) F 1 ( y 0 ) | < ε ,
de donde hemos demostrado que F 1 es continua en y 0 , como se desee.

¡Lo pensaré! Si puedo encontrar una prueba ingeniosa y directa, la agregaré a mi respuesta.

Llamar B = F ( R ) y deja gramo : B R ser F es inversa. Tenga en cuenta que gramo también es estrictamente creciente. Suponer gramo es discontinua en un punto y B . Dejar α = límite w B , w y gramo ( w ) y β = límite z B , z y + gramo ( z ) . Desde gramo es creciente y discontinuo en y , tenemos α < β .

Llevar X 0 [ α + β α 3 , β β α 3 ] .

Entonces, para lo suficientemente pequeño d > 0 , sucede que gramo ( y d ) < X 0 < gramo ( y + d ) . Tenga en cuenta que d no depende de X 0 .

Aplicar F a la desigualdad anterior se obtiene y d < F ( X 0 ) < y + d . Alquiler d 0 , encontramos eso F ( X 0 ) = y . Entonces F es en realidad constante en el intervalo no degenerado [ α + β α 3 , β β α 3 ] !!

Aquí supuse y se puede aproximar con puntos de B de ambos lados. Pero se puede usar el mismo argumento si y es sólo un punto límite izquierdo o un punto límite derecho de B , para hacer esto solo necesitas usar gramo ( y ) en lugar de β y α , respectivamente.

¡Buen argumento!

Primero me gustaría agradecer a @Cameron Buie por su esfuerzo para señalar la falla en la prueba OP.

Por supuesto que la afirmación es falsa. La monotonicidad implica continuidad en casi todas partes. Incluso una función estrictamente monótona no es necesariamente continua. Tampoco lo es su inversa, que también es estrictamente monótona.

He aquí un contraejemplo simple:

F ( X ) = { X + 2 , X > 0 ; 1 , X = 0 ; X , X < 0 .
La función F : R ] , 0 [ { 1 } ] 2 , [   es una biyección estrictamente creciente.

El inverso estrictamente creciente F 1 : ] , 0 [ { 1 } ] 2 , [ R

F 1 ( X ) = { X 2 , X > 2 ; 0 , X = 1 ; X , X < 0 .
es discontinuo

Me parece que 2 < 0 pero F ( 2 ) = 2 > 1 = F ( 0 ) entonces F no es estrictamente creciente
el error tipográfico X se corrige a X .
Dejar gramo = F 1 : entonces gramo es claramente cierto para X < 0 y también límite X 0 = 0 = límite X 0 + gramo ( X ) (Es vagamente cierto que el límite de la derecha existe y es igual a 0 ) por lo que también es continua en 0 ; también es vagamente cierto que límite X 1 gramo ( X ) = límite X 1 + gramo ( X ) = gramo ( 1 ) = 0 (tomar cualquiera ε > 0 y d = 1 2 ); es claramente continua para X > 2 y límite X 2 gramo ( X ) = límite X 2 + gramo ( X ) = gramo ( 2 ) = 0 entonces gramo = F 1 es de hecho continuo.
@lorenzo. No, porque los límites límite X 0 + gramo y límite X 2 gramo no existe. Déjame probarte la discontinuidad de otra manera. ¿Cuál es la imagen inversa del intervalo abierto? ] 1 / 2 , 3 / 2 [ ? es el conjunto { 1 } ] 5 / 2 , 7 / 2 [ que no es un conjunto abierto. Convencido ?